Download as pdf or txt
Download as pdf or txt
You are on page 1of 27

C H A P T E R

4
Moving Charges
and Magnetism
A Quick Recapitulation of the Chapter
1. The space in the surroundings of a magnet or a 6. KEmax of charged particle accelerated by cyclotron is
current-carrying conductor in which its magnetic q 2B 2R 2
influence can be experienced is called magnetic KEmax =
2m
field. Its SI unit is Tesla (T).
where, R = radius of circular track of charged particle.
2. Lorentz force is the total force F experienced
by charge (q) when it is moving with velocity v 7. Biot-Savart’s law According to this law, the magnetic
in the presence of electric field E and magnetic field due to small current-carrying element ldl at any
field B. nearby point is given by
F = q ( v × B + E) (vector form) I dB
Magnitude of magnetic force, F = qvB sin θ and
dl r
direction of force is given by right hand palm rule or P
Fleming’s left hand rule.
3. Force experienced by a current-carrying conductor
having current I and length l, when placed in a µ 0 Idlr$
dB = ⋅
magnetic field B is F = l × B = lB sin θ 4π |r | 2
4. When charged particle enters into a magnetic field µ 0 Idl sin θ
or dB = ⋅
perpendicularly, then 4π r2
mv 2 mv
(i) = qvB (ii) r = 8. Magnetic field at any point along the axis of circular
r qB current-carrying conductor is
2πm qB
(iii) T = (iv) ν = µ 0Ia 2
qB 2 πm B=
2 (r + a 2 ) 3 / 2
2
q 2B 2r 2
(v) KE =
2m
5. The frequency of a charged particle (q) when it √r 2 + a 2
a
enters perpendicularly into the magnetic field B and it P
qB
attains a circular path, is ν = r
2 πm
where, m is the mass of charged particle. This is called M
cyclotron frequency.
9. Magnetic field at the centre of a circular 13. Magnetic field due to toroidal solenoid
current-carrying conductor/ coil (i) Inside the toroidal solenoid,
µ I B = µ 0nI
B= 0
2r
r

r
I
I I

where, r is the radius of a circular loop. N


Here, n= ,
10. Ampere’s circuital law The line integral of the 2πr
magnetic field B around any closed loop is equal to N = total number of turns
µ 0 times the total current I flowing through the loop, (ii) In the open space, exterior of toroidal solenoid,

I B =0
B 14. Force between the two parallel current-carrying
conductors is

Magnetic
I field lines I1 I2
r
i.e., ∫ B ⋅ d l = µ 0I

Magnitude of magnetic field of a straight wire using µ 0 2 I1 I 2


µ I F = ⋅ .L
Ampere’s law, B = 0 4π r
2 πr If the direction of flow of current is same, then they
11. Magnetic field due to straight current-carrying attract each other otherwise they will repel each other.
conductor at any point P at a distance r from the wire 15. Torque on a current loop,
is given by
τ=M×B=NI AB
where, M is the magnetic moment.
r
P 16. Magnetic moment, M = N I A
Long straight wire
where, A is the area and I is the current.
17. The magnetic field due to current in a circular current
µ 0 2I loop as magnetic dipole, is
B= ⋅
4π r µ 2 π r 2i
1 B= 0
⇒ B∝ 4π x 3
r Here, x is the distance along the axis from the centre of
12. Magnetic field due to a straight solenoid the loop.
µ e
18. The ratio l = is called the gyromagnetic ratio
l 2me
and is constant. Its value is 8.8 × 1010 C / kg for an
electron.
φ NAB
19. Current sensitivity, =
(i) At any point inside the solenoid, B = µ 0nI I k
where, n = number of turns per unit length. where, φ is deflection produced and k is spring
1 constant.
(ii) At the ends of the solenoid, B = µ 0nI φ NAB
2 20. Voltage sensitivity, =
V kR
Objective Questions Based on NCERT Text

Topic 1
Magnetic Force
1. If a proton is projected in a direction perpendicular to 6. When a charged particle moves in the region of
a uniform magnetic field with velocity v and an magnetic field, then
electron is projected along the lines of force, what (a) magnitude of its velocity keeps on changing
will happen to proton and electron? (b) velocity of particle remains constant
(a) The electron will travel along a circle with constant (c) direction of momentum keeps on changing
speed and the proton will move along a straight line (d) kinetic energy of particle keeps on changing
(b) Proton will move in a circle with constant speed and
there will be no effect on the motion of electron 7. A proton enters a magnetic field of flux density
(c) There will not be any effect on the motion of electron 1.5 Wbm −2 with a velocity of 2 × 10 7 ms −1 at an
and proton angle of 30° with the field. The force on the proton
(d) The electron and proton both will follow the path of a will be
parabola (a) 2.4 × 10−12 N
2. Which of the following diagrams is correct? (b) 0.24 × 10−12 N
(c) 24 × 10−12 N
F (d) 0.024 × 10−12 N
(a) θ
B (b) v 8. An ionised gas contains both positive and negative
θ – B
B ions. If it is subjected simultaneously to an electric
F v
field along the + x-direction and a magnetic field
along the + z-direction, then
B (a) positive ions deflect towards + y-direction and negative
(c) F
(d) F B ions towards − y-direction
v θ v (b) all ions deflect towards + y-direction
θ – (c) all ions deflect towards − y-direction
(d) positive ions deflect towards − y-direction and
3. Magnetic field produced by a wire carrying current negative ions towards + y-direction
(a) is parallel along the axis of wire
9. A current-carrying wire of area A, length l, number of
(b) is perpendicular to the plane of the wire
density of charge carriers n is placed in a region of
(c) forms circular loops along the axis of wire and coplanar
to the wire external magnetic field B. What will be net force on
(d) direction of field is not constant charged carriers?
(a) ( nAl )q ( vd × B ) (b) q ( vd × B )
4. An electron is travelling horizontally towards East. A
(c) nq ( vd × B ) (d) Alq ( vd × B )
magnetic field in vertically downward direction exerts
a force on the electron along 10. A current-carrying closed loop in the form of a right
(a) East (b) West angled isosceles ∆ ABC is placed in a uniform
(c) North (d) South magnetic field acting along AB. If the magnetic force
5. If a particle of charge 10 −12 C moving along the on the arm BC is F, then force on the arm AC is
$
X-direction with a velocity 10 5 ms −1 experiences a A

force of 10 −10 N in Y-direction


$ due to magnetic field,
then the minimum magnetic field is
(a) 6.25 × 103 T in Z-direction
$
−15 $
(b) 10 T in Z-direction
(c) 6.25 × 10−3 T in Z-direction
$ B C
−3 $
(d) 10 T in Z-direction (a) −F (b) F (c) 2 F (d) − 2 F
11. A square current-carrying loop is suspended in a 16. A particle of mass m and charge q moves with a
uniform magnetic field acting in the plane of the constant velocity v along the positive x-direction. It
loop. If the force on one arm of the loop is F, enters a region containing a uniform magnetic field B
the net force on the remaining three arms of the directed along the negative z-direction, extending
loop is from x = a to x = b. The minimum value of v required
(a) 3 F (b) – F so that the particle can just enter the region x > b is
(c) –3 F (d) F (a) qb B / m (b) q ( b − a ) B / m
12. An electron enters a region of magnetic field (c) q aB / m (d) q ( b + a ) B / 2m
perpendicularly with a speed of 3 × 10 7 ms −1 . 17. An α-particle with a specific charge of 2.5 × 10 7 C/kg
Strength of magnetic field is 6 × 10 −4 T. Frequency moves with a speed of 2 × 10 5 ms −1 in a perpendicular
and energy of electron respectively are magnetic field of 0.05 T. The radius of circular path
(a) 2 MHz, 25 keV described by the particle is
(a) 8 cm (b) 4 cm (c) 16 cm (d) 2 cm
(b) 20 MHz, 2.5 keV
(c) 20 MHz, 25 keV 18. A proton of energy 1 MeV describes a circular path in
(d) 2 MHz, 2.5 keV plane at right angles to a uniform magnetic field of
6.28 × 10 −4 T. The mass of proton is1.7 × 10 −27 kg.
13. Which one is a correct figure to represent path of a
The frequency of proton is nearly equal to
moving charge in magnetic field?
(a) 107 Hz (b) 105 Hz
Y B Y (c) 106 Hz (d) 104 Hz
19. A proton of mass 1.67 × 10 −27 kg and charge
(a) (b) 1.6 × 10 −19 C is projected with a speed of 2 × 10 6 ms −1
v at an angle of 60° to the X -axis. If a magnetic field of
v
X X 0.104 T is applied along Y -axis, then path of proton is
O O (a) a circle of radius 0.2 m and time period of π × 10−7 s
(b) a circle of radius 0.1 m and time period 2π × 10−7 s
Y B Y
B (c) a helix of radius 0.2 m and time period of 2π × 10−7 s

(c) (d) (d) a helix of radius 0.2 m and time period of 4 π × 10−7 s
v v 20. A straight wire of mass 200 g and length 1.5 m carries
X X a current of 2 A. It is held in mid-air by a uniform
O O
horizontal magnetic field. What is the magnitude of
14. A proton, a deuteron and an α-particle enter a region magnetic field?
of perpendicular magnetic field (to their velocities) (a) 1 T (b) 0.65 T (c) 0.35 T (d) 0.85 T
with same kinetic energy. If rp , rd and rα are the radii 21. An electron moves in a circular orbit with a uniform
of circular paths of these particles, then speed v. It produces a magnetic field B at the centre of
(a) rα = rd < rp the circle. The radius of the circle is proportional to
(b) rα = rd = rp B v
(a) (b)
(c) rα < rd < rp v B
(d) rα > rd > rp v B
(c) (d)
15. Two particles of masses ma and mb with same charge B v
are projected in a perpendicular magnetic field. They 22. A deuteron of kinetic energy 50 keV is describing a
travel along circular path of radii ra and rb such that circular orbit of radius 0.5 m in a plane perpendicular
ra > rb . Which is correct? to magnetic field B. The kinetic energy of the proton
(a) ma va > mb vb that describes a circular orbit of radius 0.5 m in the
(b) ma > mb and va > vb same plane with the same magnetic field B is
(c) ma = mb and va > vb (a) 25 keV (b) 50 keV
(d) mb vb > ma va (c) 200 keV (d) 100 keV
Topic 2
Motion in Combined Electric and Magnetic Field
23. In the given figure, E and B are Y 28. Cyclotron frequency of an electron circulating in a
magnitudes of electric and E magnetic field of 1T is
magnetic fields which are acting v (a) 28 MHz (b) 280 MHz (c) 2.8 GHz (d) 28 GHz
on a moving electron as shown. – X
B 29. An electron is moving in a cyclotron at a speed of
The direction of velocity of
3.2 × 10 7 ms −1 in a magnetic field of 5 × 10 −4 T
electron is along X -axis. Z
Then, force on electron is given by perpendicular to it. What is the frequency of this
electron? ( q = 1.6 × 10 −19 C, me = 9.1 × 10 −31 kg )
(a) F = q ( E + vB ) $j (b) F = q ( E − vB ) $j
(a) 1.4 × 105 Hz (b) 1.4 × 107 Hz
(c) F = q ( E + vB ) k$ (d) F = q ( E − vB ) k$
(c) 1.4 × 106 Hz (d) 1.4 × 109 Hz
24. An electron moves straight σ
inside a charged parallel plate 30. Which of the following figures correctly depicts the
capacitor. Plates of capacitor direction of magnetic field of a current-carrying coil?
σ
have charge density σ. The space l
between plates is filled with I I
constant magnetic field of induction B. (a) (b)
Time taken by electron to pass through the region is
eσ ε 0 lB σe ε0 B
(a) (b) (c) (d)
ε 0 lB σ ε0 B eσ
I I
25. A cyclotron is used to accelerate charged particles or (c) (d)
ions to high energies. It uses
(a) only electric field
(b) only magnetic field
(c) Both electric and magnetic fields 31. A tightly wound coil of 100 turns with radius
(d) None of the above 10 cm, magnitude of carrying-current in coil is 1 A,
what will be the magnitude of magnetic field at
26. An alternating electric field of frequency ν, is applied centre of coil?
across the dees (radius = R ) of a cyclotron that is (a) 6.28 × 10− 6 T (b) 6.28 × 10− 7 T
being used to accelerate protons (mass = m). The (c) 6.28 × 10− 5 T (d) 6.28 × 10− 4 T
operating magnetic field ( B ) used in the cyclotron and
the kinetic energy ( K ) of the proton beam produced 32. A copper coil of 100 turns, radius 8.0 × 10 − 2 m
by it, are given by [CBSE AIPMT 2012] carries a current of 0.40 A. Magnitude of magnetic
mν field at centre of coil is
(a) B = and K = 2mπ 2 ν 2 R 2
e (a) B = 3.1 × 10− 3 T (b) B = 3.1 × 10− 2 T
2πmν (c) B = 3.1 × 10− 4 T (d) B = 3.1 × 10− 7 T
(b) B = and K = m2 πνR 2
e
2πmν 33. Two concentric circular coils X and Y of radii 16 cm
(c) B = and K = 2mπ 2 ν 2 R 2
e and 10 cm lie in same vertical plane containing North
mν to South direction. Coil X has 20 turns, coil Y has
(d) B = and K = m2 πνR 2 25 turns and current in coil X is16 A whereas in coil Y
e
is18 A. Current in X is anti-clockwise and in Y is
27. A proton and an α-particle both enter a region of clockwise. For an observer facing West and looking at
uniform magnetic field B, moving at right angles to coils, magnetic field at the centre of assembly of coils is
the field B. If the radius of circular orbits for both the (a) 1.57 × 10− 3 T towards East
particles is equal and the kinetic energy acquired by
proton is 1 MeV, the energy acquired by the (b) 1.57 × 10− 3 T towards West
α-particle will be [CBSE AIPMT 2015] (c) 1.57 × 10− 3 T towards North
(a) 4 MeV (b) 0.5 MeV (c) 1.5 MeV (d) 1 MeV (d) 15.7 × 10− 3 T towards South
34. A conducting loop carrying a current I is placed in a 35. A straight long wire carries a current of 35 A. Its
uniform magnetic field pointing into the plane of the magnitude of magnetic field at a distance of 0.20 m
paper as shown in figure. from the wire is
Y
The loop will have a tendency to I Fm
(a) 3.5 × 10− 5 T
(a) contract (b) 3.5 × 105 T
(b) expand X (c) 3.5 T
(c) move towards positive X -axis (d) 7T
(d) move towards negative X -axis

Topic 3
Biot-Savart’s Law and Ampere’s Law
36. Vector form of Biot-Savart’s law is 40. If a current loop of radius R carrying a anti-clockwise
µ l × dl I dl × r current I is placed in a plane parallel to YZ-plane,
(a) dB = 0 (b) d B =
4 π r2 r3 then magnetic field at a point on the axis of the loop is
µ I dl × r µ I dl × r given by
(c) dB = 0 (d) dB = 0
4π r3 4π r2 µ 0 IR 2
(a) B = $j
37. An element ∆I = ∆x $i is placed at the origin and carries 2 ( x 2 + R 2 )3 / 2
a current I =10 A. µ 0 IR 2
(b) B = k$
Y 2 ( x 2 + R 2 )3 / 2
µ 0 IR 2
P (c) B = i$
2 ( x 2 + R 2 )3 / 2
0.5 m µ 0 IR 2
(d) B = ( $i × k$ )
2 ( x 2 + R 2 )3 / 2
X
∆x 41. An infinitely long conductor PQR is bent to form a
If ∆x =1 cm, magnetic field at point P is right angle as shown. A current I flows through
(a) 4 × 10− 8 k$ T PQR. The magnetic field due to this current at the
(b) 4 × 10− 8 i$ T point M is H1 . Now, another infinitely long straight
conductor QS is connected at Q, so that the current
(c) 4 × 10− 8 $j T
is 1/2 in QR as well as in QS. The current in PQ
(d) − 4 × 10− 8 $j T remaining unchanged. The magnetic field at M is
now H 2 . The ratio H1 / H 2 is given by
38. There is a thin conducting wire carrying current. What
is the value of magnetic field induction at any point on M
the conductor itself?
(a) 1 (b) Zero 90°
–¥ +¥
(c) − 1 (d) Either (a) or (b) P I Q S

39. A straight wire carrying a current of 12 A is sent into a R –¥


semicircular loop of radius 2.0 cm. What will be 1 2
(a) (b) 1 (c) (d) 2
magnetic field at the centre of the semicircular loop? 2 3

42. A polygon shaped wire is inscribed in a circle of


I I radius R. The magnetic induction at the centre of
polygon, when current flows through the wire is
(a) B = 1.9 × 10− 4 T µ 0n I  2π  µ 0n I  4π 
(a) tan   (b) tan  
(b) B = 1.9 × 10 T
4
2πR  n 2πR  n
(c) B = 1.9 × 10− 4 T, ⊗ µ 0n I  π µ 0n I  π
(c) tan   (d) tan  2 
(d) B = 1.9 × 10 4 T, ⊗ 2πR  n 2πR n 
43. Two identical wires A and B, each of length l, carry 48. A solenoid of length 0.5 m has a radius of 1 cm and is
the same current I. Wire A is bent into a circle of made up of 500 turns. It carries a current of 5A. What
radius R and wire B is bent form a square of side a. If is the magnetic field inside the solenoid?
B A and B B are the values of magnetic field at the (a) 6.28 × 10−3 T (b) 6.28 × 103 T
centres of the circle and square respectively, then the (c) 6.28 × 10− 5 T (d) 6.28 × 105 T
ratio B A / B B is [JEE Main 2016]
π2 π2 49. A long solenoid has 20 turns cm −1 . The current
(a) (b) necessary to produce a magnetic field of 20 mT inside
8 16 2
the solenoid is approximately
π2 π2 (a) 1A (b) 2 A (c) 4 A (d) 8 A
(c) (d)
16 8 2
50. For a toroid, magnetic field strength in the region
44. The magnetic field at the centre of a circular current enclosed by wire turns is given by
carrying-conductor of radius r is B c . The magnetic (a) B = µ 0 n I, where n = number of turns
field on its axis at a distance r from the centre is B a . (b) B = µ 0 I / n , n = number of turns per metre
The value of B c : B a will be µ I
(c) B = 0 , r = mean radius
(a) 1 : 2 (b) 1 : 2 2 2r
µ 0 NI  N = number of turns
(c) 2 2 : 1 (d) 2 :1 (d) B = ,
2πr  r = radius of toroid
45. A long straight wire of radius a carries a steady
current I. The current is uniformly distributed over its 51. A toroid of core of inner radius 0.25 m and outer
cross-section. The ratio of the magnetic fields B and radius 0.26 m around which 3500 turns of a wire are
a wound. If the current in the wire is 11 A, then
B′ at radial distances and 2a respectively, from the magnetic field inside the core of the toroid is
2
axis of the wire is [NEET 2016]
(a) 3 × 102 T (b) 3 × 10−2 T (c) 3 × 10−7 T (d) 3 × 107 T
1 1 52. If two parallel current-carrying conductors placed 1 m
(a) (b) 1 (c) 4 (d)
2 4 apart in vacuum are placed such that each carries 1 A
current, then there is a force of
46. For a cylindrical conductor of radius a, which of the
following graphs shows a correct relationship of B (a) 2 × 10− 7 N per metre of length
versus r? (b) 2 × 107 N per metre of length
B B (c) 9 × 109 N per metre of length
(d) 9 × 10− 9 N per metre of length
(a) (b) 53. Two identical long conducting wires AOB and COD
are placed at right angle to each other, with one above
r r
r=a r=a other such that O is their common point for the two.
B B The wires carry I 1 and I 2 currents, respectively. Point
P is lying at distance d from O along a direction
(c) (d) perpendicular to the plane containing the wires. The
magnetic field at the point P will be [CBSE AIPMT 2014]
r r  I1 
a r=a µ0 µ0
(a)   (b) ( I1 + I 2 )
2πd  I2  2πd
47. Which of the following represent a correct figure to
µ µ
display of magnetic field lines due to a solenoid? (c) 0 ( I12 − I 22 ) (d) 0 ( I12 + I 22 )1 / 2
2πd 2πd
Cross-section of wires Cross-section of wires
54. Two long parallel straight wires A and B carrying
(a) (b) currents of 4.0 A and 5.0 A in same direction
separated by a distance of 4 × 10 − 2 m.
The force on a 0.20 m section of wire A is
Cross-section of wires
Cross-section of wires (a) 2 × 10− 5 N towards B
(c) (d) (b) 2 × 10− 5 N away from B
(c) 2 × 10− 5 N perpendicular to B
(d) 2 × 10− 5 N parallel to B
55. Two very long, straight and parallel wires carry 58. Two thin long parallel wires separated by a distance d
steady currents I and I, respectively. The distance carry a current of I ampere in same direction. They will
between the wires is d. At a certain instant of time µ 0I 2
a point charge q is at a point equidistant from the (a) attract each other with a force of
2πd
two wires in the plane of the wires. Its instantaneous
velocity v is perpendicular to this plane. µ 0I 2
(b) repel each other with a force of
The magnitude of the force due to the magnetic field 2πd
acting on the charge at their instant is µ 0I 2
(c) attract each other with a force of
µ 0Iqv µ 0Iqv 2µ 0Iqv 2πd 2
(a) (b) (c) (d) 0
2πd πd πd µ 0I 2
(d) repel each other with a force of
56. A square loop ABCD Y
B C 2πd 2
carrying a current i, is 59. Three long, straight and parallel wires carrying
placed near and coplanar I i L currents are arranged as shown in the figure.
with a long straight
conductor XY carrying a
X A D
current I, the net force on 15 A 50 A 10 A
L/2 L
the loop will be
µ 0 Ii 2µ 0 IiL µ 0 IiL 2µ 0 Ii x (15 – x)
(a) (b) (c) (d)
2π 3π 2π 3π
57. The wire which connects the battery of a car to its
The wire C which carries a current of 50 A is so
starter motor carries current of 300 A during
replaced that it experiences no force. The distance of
starting. Force per unit length between wires
wire C from wire A is
(wires are 0.7 m long and 0.015 m distant apart) is (a) 9 cm (b) 7 cm
(a) 1.2 Nm− 1 repulsive (b) 1.2 Nm− 1 attractive (c) 5 cm (d) 3 cm
(c) 2.4 Nm− 1 repulsive (d) 2.4 Nm− 1 attractive

Topic 4
Torque on Current Loop and Magnetic Dipole
60. A rectangular loop of sides 10 cm and 5 cm carrying 61. A current-carrying loop is placed in a uniform
a current I of 12 A is placed in different orientations magnetic field in four different orientations, I, II, III
as shown in the figures. and IV. Arrange them in the decreasing order of
z potential energy.
I z
B
I I B
(a) y (b) ^
n
I I I. B II. B
I y
x I
z x I ^
z n
I B
I B ^
n
(c) I y (d)
I I
I y
x I III. B IV. B
x I
^
n
If there is a uniform magnetic field of 0.3 T in the
positive z-direction in which orientations the loop (a) I > III > II > IV
would be in (i) stable equilibrium and (ii) unstable (b) I > II > III > IV
equilibrium? [JEE Main 2015] (c) I > IV > II > III
(a) (a) and (b) respectively (b) (a) and (c) respectively (d) III > IV > I > II
(c) (b) and (d) respectively (d) (b) and (c) respectively
62. A steady current i flows in a small square loop of wire 68. Two similar coils are placed mutually perpendicular
of side L in a horizontal plane. The loop is now folded such that their centres coincide. At centre, the ratio of
about its middle such that half of it lies in a vertical the magnetic field due to one coil and the resultant
plane. Let µ 1 and µ 2 respectively denote the magnetic magnetic field of both coils for same current will be
moments due to the current loop before and after (a) 1 : 2 (b) 1 : 2 (c) 2 : 1 (d) 3 :1
folding. Then,
(a) µ 2 = 0 69. Magnetic moment (or magnetic dipole moment) of a
(b) µ 1 and µ 2 are in the same direction current-carrying coil is given by
| µ1 | I
(c) = 2 (a) m = INA (b) m =
|µ2 | A
NI
|µ 1 |  1  (c) m = NA (d) m =
(d) =  A
|µ 2 |  2 
70. The magnitude of the magnetic field at the centre of
63. A coil in the shape of an equilateral triangle of side l the tightly wound 150 turns coil of radius 12 cm
is suspended between the pole pieces of a permanent
carrying a current of 2A, is
magnet such that B is in plane of the coil. If due to a
(a) 18 G (b) 19.7 G
current i in the triangle a torque τ acts on it, the side l
(c) 15.7 G (d) 17.7 G
of the triangle is
2  τ
1/ 2
2  τ 71. A loop of flexible wire of irregular shape carrying
(a)   (b)   current is placed in an external field. Then,
3  Bi  3  Bi 
1/ 2
(a) it rotates in a direction perpendicular to its axis
 τ  1 τ (b) it rotates along an axis perpendicular to its plane
(c) 2   (d)
 3 Bi  3 Bi (c) it does not show any change
(d) it assumes a circular shape
64. Torque on a current-carrying loop of magnetic
moment m, placed in region of magnetic field B is 72. A circular current loop of magnetic moment M is in
given by an arbitrary orientation in an external magnetic field
1 B. The work done to rotate the loop by 90° about an
(a) τ = m × B (b) = µ 0 m × B axis perpendicular to its plane is
2
µ0 3
(c) τ = (m × B ) (d) τ = m × B (a) MB (b) MB
4π 2
MB
65. A current loop in a magnetic field [NEET 2013] (c) (d) zero
2
(a) experiences a torque whether the field is uniform or
non-uniform in all orientations 73. Magnetic field at the centre of a circular loop of area
(b) can be in equilibrium in one orientations A is B. The magnetic moment of the loop will be
(c) can be equilibrium in two orientations, both the BA 2 BA 3 / 2
equilibrium states are unstable (a) (b)
µ 0π µ 0π
(d) can be in equilibrium in two orientations, one stable
while the other is unstable BA 3 / 2 2BA 3 / 2
(c) (d)
µ 0 π1 / 2 µ 0 π1 / 2
66. A circular coil of 25 turns and radius 12 cm is placed
in a uniform magnetic field of 0.5 T normal to the 74. For hydrogen atom, which of the following figure
plane of the coil. If the current in the coil is 6 A, then correctly, shows the direction of magnetic moment
total torque acting on the coil is related to revolving electron?
(a) zero (b) 3.4 N m (c) 3.8 N m (d) 4.4 N m e– e–

67. For a circular current loop magnitude of magnetic field


(a) +Ze µe (b) +Ze µe
on axis of loop at a point distant x from centre of loop
(x >> R and magnetic dipole moment of loop is m = IA)
is e– e–
µ0 m µ m
(a) B = (b) B = 0 3
x3 4π x (c) +Ze (d) +Ze
µ 2m µ0 m⋅R µe µe
(c) B = 0 3 (d) B =
4π x 2π x 3
75. In a hydrogen atom, an electron revolves around the 77. The value of Bohr magneton is
nucleus in an orbit of radius r with velocity v, the (a) 9.27 × 10−24 Am2 (b) 9.27 × 10−27 Am2
current corresponding to the revolving electron is
er ev ev ev (c) 8.8 × 10−10 Am2 (d) 6.25 × 10−12 Am2
(a) I = (b) I = (c) I = (d) I =
2πv 2π 2πr πr 78. Magnitude of angular magnetic moment
76. In a hydrogen atom, an electron moves in a circular associated with a revolving electron in a hydrogen
−11 atom will be
orbit of radius 5.2 × 10 m and produces a magnetic
induction of 12.56 T at its centre. The current e −el 2
(a) µ l = (b) µ l =
produced by the motion of the electron will be 2me 2me
(a) 6.53 × 10−3 A (b) 13.25 × 10−10 A −e − el
(c) µ l = (d) µ l =
(c) 9.6 × 106 A (d) 1.04 × 10−3 A 4 me 2me

Topic 5
Moving Coil Galvanometer
79. A moving coil galvanometer is an instrument which 85. In ballistic galvanometer, the frame on which the coil
(a) is used to measure EMF is wound is non-metallic to
(b) is used to measure potential difference (a) avoid the production of induced emf
(c) is used to measure resistance (b) avoid the production of eddy current
(d) is a deflection instrument which gives a deflection (c) increase the production of eddy current
when a current flows through its coil (d) increase the production of induced emf
80. In a moving coil galvanometer of coil of N -turns of 86. Current sensitivity of a galvanometer is
area A have a spring of stiffness k. NBA k NBA kR
(a) (b) (c) (d)
If coil is deflected by some angle φ due to flow of I k NBA kR NBA
current in uniform radial magnetic field B, then
87. To increase the current sensitivity of a moving coil
 NAB   k 
(a) φ =  I (b) φ =  I galvanometer, we should decrease
 k   BNA  (a) strength of magnet (b) torsional constant of spring
 kA   BN  (c) number of turns in coil (d) area of coil
(c) φ =  I (d) φ =  I
 BN   kA  88. The coil of a galvanometer consists of 100 turns and
81. To make the field radial in a moving coil galvanometer effective area of 1 cm 2 . The restoring couple is
(a) number of turns of coil is kept small 10 −8 Nm rad −1 . The magnetic field between poles is
(b) magnet is taken in the form of horse-shoe of 5 T. Current sensitivity of this galvanometer is
(c) poles are of very strong magnets (a) 5 × 104 rad /µ amp (b) 5 × 106 per amp
(d) poles are cylindrically cut
(c) 2 × 10−7 per amp (d) 5rad /µ amp
82. The deflection in a moving coil galvanometer is
(a) directly proportional to torsional constant of spring 89. A galvanometer has a coil of resistance15 Ω and meter
(b) directly proportional to the number of turns in the coil shows full scale deflection for a current of 4 mA. To
(c) inversely proportional to the area of the coil convert it into an ammeter of 0 to 6 A range,
(d) inversely proportional to the current in the coil (a) a resistance of 10 mΩ is required in series
83. In a moving coil galvanometer having a coil of (b) a resistance of 10 mΩ is required in parallel
N -turns of area A and carrying current I is placed in a (c) a resistance of 1000 Ω is required in series
radial field of strength B. (d) a resistance of 1000 Ω is required in parallel
The torque acting on the coil is 90. In two galvanometers A and B, to produce a
2 2
(a) NA B I (b) NABI 2 2
(c) N ABI (d) NABI deflection of 10 divisions, currents of 3 mA and 5 mA
are required respectively, then
84. What is the shape of magnet used in moving coil (a) A is more sensitive than B
galvanometer to make the magnetic field radial (b) B is more sensitive than A
(a) concave (b) horse-shoe magnet (c) both are equally sensitive
(c) convex (d) None of these (d) sensitivity of B is 5 / 3 that A
91. For the given ammeter circuit, 96. To convert a galvanometer into a voltmeter,
SΩ (a) a low resistance in parallel is used
(b) a low resistance in series is used
(c) a high resistance in series is used
I Ig (d) a high resistance in parallel is used
G
GΩ
97. For the voltmeter circuit given,
(a) I g S = IG (b) ( I − I g )S = I g G
S
I G G
(c) I g G = ( I + I g )S (d) =
Ig S Ig
RL
92. To convert a galvanometer into an ammeter, I
(a) a low resistance is connected in series with the coil of Ig G I R +G
galvanometer (a) = (b) = L
(b) a low resistance is connected in parallel with the coil I S Ig S
of galvanometer (c) ( I − I g )RL = I g (G + S ) (d) IRL = I g G
(c) a high resistance is connected in series with
galvanometer coil 98. A galvanometer with a coil of resistance 12 Ω shows
(d) a high resistance is connected in parallel with the full scale deflection for a current of 2.5 mA. The ratio
galvanometer coil of net resistance of an ammeter of range 0 to 7.5 A and
93. A galvanometer of resistance 70 Ω, is converted into a voltmeter of range 0 to 10 V is
an ammeter by a shunt resistance rs = 003
. Ω. The (a) 10−12 (b) 10−7 (c) 10−6 (d) 10−8
value of its resistance will become 99. A galvanometer coil has a resistance of12 Ω and meter
(a) 0.025 Ω (b) 0.022 Ω (c) 0.035 Ω (d) 0.030 Ω shows full scale deflection for a current of 3 mA. To
convert galvanometer into a voltmeter of range 0 to 18 V,
94. To increase current sensitivity of a moving coil
(a) a resistance of 5988 Ω in series is required
galvanometer by 50%, its resistance is increased so
(b) a resistance of 5988 Ω in parallel is required
that its new resistance is twice of its initial resistance.
(c) a resistance of 5.988 Ω in series is required
Its voltage sensitivity changes by a factor of
(d) a resistance of 5.988 Ω in parallel is required
(a) increases by a factor of 2
(b) decreases by a factor of 1 / 2 100. Relation between voltage sensitivity ( S V ) and current
(c) increases by a factor of 4 sensitivity ( S i ) for a galvanometer of resistance G
(d) decreases by a factor of 1/ 4 ohms is
Si SV
(a) = SV (b) = Si
95. Voltage sensitivity of a galvanometer is G G
NBA k NBA kR (c) S i S V = G (d) S iSV = G
(a) (b) (c) (d)
k NBA kR NBA

Special Format Questions


I. Assertion and Reason 101. Assertion Cyclotron is a device which is used to
■ Directions (Q. Nos. 101-105) In the following accelerate the positive ions.
questions, a statement of assertion is followed by a Reason Cyclotron frequency depends upon the velocity.
corresponding statement of reason. Of the following 102. Assertion If a proton and an α-particle enter a uniform
statements, choose the correct one. magnetic field perpendicularly with the same speed, the
(a) Both Assertion and Reason are correct and Reason time period of revolution of α-particle is double than
is the correct explanation of Assertion. that of proton.
(b) Both Assertion and Reason are correct but Reason Reason In a magnetic field, the period of revolution at
is not the correct explanation of Assertion. a charged particle is directly proportional to the mass of
(c) Assertion is correct but Reason is incorrect. the particle and inversely proportional to the charge of
(d) Assertion is incorrect but Reason is correct. particle.
103. Assertion The ion cannot move with a speed beyond Statement II The electron is an elementary particle,
a certain limit in a cyclotron. it does not spin around an axis and it does not possess
any intrinsic magnetic moment.
Reason As velocity increases, then time taken by ion
increases. 110. Statement I If we increase the current sensitivity of
a galvanometer by increasing number of turns, its
104. Assertion The magnetic field produced by a
voltage sensitivity also increases.
current-carrying solenoid is independent of its length
and cross-sectional area. Statement II Resistance of a wire is directly
proportional to its length.
Reason The magnetic field inside the solenoid is
uniform.
Statement Based Questions (Type II)
105. Assertion An electron and a proton enter a magnetic
field with equal velocities, then the force experienced 111. Consider a moving charged particle in a region of
by the proton will be more than electron. magnetic field. Which of the following are correct?
Reason The mass of proton is 1837 times more than I. If vis parallel to B, then path of particle is spiral.
that of electron. II. If v is perpendicular to B, then path of particle is a circle.
III. Ifv hasacomponentalongB,thenpathofparticleishelical.
II. Statement Based Questions (Type I) IV. If vis along B, then path of particle is a circle.
(a) I and II (b) II and III
■ Directions (Q. Nos. 106-110) In the following (c) III and IV (d) IV and I
questions, a statement I is followed by a corresponding
statement II. Of the following statements, choose the 112. An electron and a proton moving on a straight parallel
correct one. paths with same velocity enter a semi-infinite region
(a) Both Statement I and Statement II are correct and of uniform magnetic field perpendicular to velocity.
Statement II is the correct explanation of Statement I. Which of these are correct?
(b) Both Statement I and Statement II are correct but
I. They will never come out of magnetic field region.
Statement II is not the correct explanation of
Statement I. II. They will come out travelling along parallel paths.
(c) Statement I is correct but Statement II is incorrect. III. They will come out same time.
(d) Statement I is incorrect but Statement II is correct. IV. They will come out at different times.
106. Statement I When a charged particle moves in the (a) I and II (b) II and III
region of a perpendicular magnetic field, its speed (c) II and IV (d) I and IV
remains constant. 113. A velocity selector; (a region of perpendicular electric
Statement II Force acting on the charged particle is and magnetic field)
given by F = q ( v × B ). I. Allows charged particles to pass straight when v = E / B.
107. Statement I When a charged particle moves in a II. Deflects particles in a direction perpendicular to both v
region of magnetic field such that its velocity is at and B, when v > E / B.
some acute angle with direction of field, its trajectory III. Deflects particles in the direction of electric field when
is a helix. v < E / B.
Statement II Perpendicular component of velocity IV. Deflects all particles in a direction perpendicular to
causes a rotating centripetal force and parallel both E and B.
component of velocity does not produces any force. (a) I, III and IV (b) II, III and IV
(c) I, II and III (d) I, II and IV
108. Statement I In cyclotron, frequency of applied
voltage (to dees) is adjusted so that ν applied voltage 114. Difference between Coulomb’s force and
= ν cyclotron = qB /2πm. This ensures acceleration of Biot-Savart’s force are
charged particle in each cycle. I. Electrostatic force is along the displacement vector
Statement II Charged particles are moved inside joining the source and the field point but magnetic
‘dee’ by magnetic field in semicircular paths and they force is perpendicular to the plane containing the
arrive in gap between the ‘dees’ in a time interval of displacement vector and source.
T sec, where T = 2πm/ Bq. II. There is an angle dependence in the Biot-Savart’s law
which is not present in Coulomb’s law.
109. Statement I Magnetic properties of iron like
III. Biot-Savart’s law can be expressed in vector form but
materials cannot be linked to intrinsic magnetic
Coulomb’s law cannot be expressed in vector form.
moment of electrons of its atoms.
IV. Biot-Savart’s law is applicable in all medium but 118. Match the following parameters of a charged particle
Coulomb’s law is applicable only in medium which moving in cyclotron with their values.
can be polarised.
(a) I and II (b) II and III Column I Column II
(c) III and IV (d) IV and I
A. Frequency of rotation of 1. B qv
115. Two coaxial solenoids 1 and 2 of the same length are charged particle ν (Hz).
set so that one is inside the other. The number of turns
per unit length are n1 and n2 . The current i1 and i2 are B. Velocity of charged particle 2. q 2B 2R 2 / 2 m
v (ms−1 ).
flowing in opposite directions. The magnetic field
inside the inner coil is zero. This is possible when C. Kinetic energy of charged 3. qBR / m
I. i1 ≠ i 2 and n1 = n 2 particle K (joule).
II. i1 = i 2 and n1 ≠ n 2 D. Force on charged particle when 4. Bq
III. i1 = i 2 and n1 = n 2 it is inside any of dees F (newton). 2 πm
IV. i1 n1 = i 2 n 2
A B C D
Which of the following statement(s) is/are correct? (a) 1 2 3 4
(a) I and II (b) II and III (b) 2 3 4 1
(c) III and IV (d) I and IV (c) 1 3 4 2
116. The galvanometer cannot as such be used as an (d) 4 3 2 1
ammeter to measure the value of current in a given 119. Match the following columns.
circuit. The following reasons are
I. galvanometergivesfullscaledeflectionforasmallcurrent. Column I Column II
II. galvanometer has a large resistance. q
III. a linear scale cannot be designed so that I ∝ φ.
A. Lorentz force 1. ∫ E ⋅ dA =
ε0
IV. a galvanometer can give inaccurate values. µ0 i dl × r
(a) I and IV (b) II and III B. Gauss’ law 2. dB = =
4π r3
(c) I and II (d) III and IV
C. Biot-Savart’s law 3. F = q(E + v × B)
III. Matching Type 1 q1q2
D. Coulomb’s law 4. F = ⋅
117. A charged particle with some initial velocity is 4 πε0 r 2
projected in a region where non-zero electric and/or
magnetic fields are present. In Column I, information A B C D A B C D
about the existence of electric and/or magnetic field (a) 3 1 2 4 (b) 1 2 4 3
and direction of initial velocity of charged particle are (c) 2 3 1 4 (d) 4 1 2 3
given, while in Column II the probable path of the 120. For a current-carrying wire loop of N-turns, placed in
charged particle is mentioned. Match the entries of region of a uniform magnetic field B, match Column I
Column I with the entries of Column II. and Column II.
Column I Column II Column I Column II
A. E = 0, B ≠ 0, and initial velocity is at 1. Straight line
any angle with B A. Torque on loop 1. mB

B. E ≠ 0, B = 0 and initial velocity is at 2. Parabola B. Torque on loop when m is either 2. NIA


any angle with E parallel or anti-parallel to B
C. E ≠ 0, B ≠ 0, E|| B and initial velocity 3. Circular C. Magnetic moment of loop 3. zero
is ⊥ to both
D. E ≠ 0, B ≠ 0, E perpendicular to B and 4. Helical path with D. Torque on loop when m is 4. m ×B
v perpendicular to both E and B non-uniform pitch perpendicular to B

A B C D A B C D
(a) 1, 3 1, 2 4 1 (a) 1 2 3 4
(b) 1,2 3,4 4 1,3 (b) 2 1 4 3
(c) 2 3 1 4 (c) 4 3 2 1
(d) 4 1 2 3 (d) 1 3 4 2
121. Figure shows the construction details of a moving coil ■ Directions (Q. Nos. 125-126) These questions are
galvanometer. based on the following situation. Choose the correct
Scale options from those given below.
The horizontal component of the earth’s magnetic field
at a certain place is 3.0 × 10 −5 T and the direction of
Pointer Permanent magnet the field is from the geographic south to the
Coil
north pole. A very long straight conductor is carrying
a steady current of 1 A.
125. What is the force per unit length on it when it is
Spring placed on a horizontal table and the direction of
N S
current is east to west?
(a) 3 × 10−5 Nm−1 (b) 5 × 10−2 Nm−1
Pivot
(c) 6 × 10−1 Nm−1 (d) 8 × 10−3 Nm−1
126. What is the force per unit length when the direction of
the current is south to north?
Pole Soft Iron core
pieces (a) 5 × 10−4 Nm−1 (b) Zero
Uniform radial
−2 −1
magnetic field (c) 6 × 10 Nm (d) 8 × 10−3 Nm−1

Match the part with its function. ■ Directions (Q. Nos. 127-128) These questions are
Column I Column II based on the following situation. Choose the correct
options from those given below.
A. Soft iron core 1. produces deflecting torque
A B
B. Pole pieces 2. produces restoring torque O a
C. Spring 3. produces radial field I1 30°
D. Coil 4. increases field strength D
b
A B C D A B C D C
(a) 1 2 3 4 (b) 1 2 4 3
(c) 2 1 3 4 (d) 4 3 2 1 The arc’s BC (radius = b) and AD (radius = a) of loop
are joined by the straight wires AB and CD. A steady
IV. Passage Based Questions current I is flowing in the loop. Angle made by AB and
CD at the origin is 30°.Another straight thin wire with
■ Directions (Q. Nos. 122-124) These questions are steady current I, flowing out of plane of the paper is
based on the following situation. Choose the correct kept at the origin.
options from those given below.
127. The magnitude of the magnetic field B due to loop
An electron with a speed v 0 << c moves in a circle of ABCD at the origin O is
radius r0 in a uniform magnetic field. The time µ 0I ( b − a )
required for one revolution of the electron is T0 . The (a) zero (b)
24 ab
speed of the electron is now doubled to 2 v 0 .
µ 0I  b − a µ I  π 
(c)   (d) 0 2( b − a ) + ( a + b )
122. The radius of the circle will change to 4 π  ab  4π  3 
(a) 4 r0 (b) 2r0 (c) r0 (d) r0 / 2
128. Due to presence of current I 1 at the origin
123. The time required for one revolution of the electron
(a) the forces on AB and DC are zero
will change to
(a) 4T0 (b) 2T0 (c) T0 (d) T0 / 2 (b) the forces on AD and BC are zero
(c) the magnitude of the net force on the loop is given by
124. A charged particle is projected in a magnetic field
µ 0 II1  π 
B = (2$i + 4$j) × 10 T. The acceleration of the particle
2
 2( b − a ) + ( a + b )
4π  3 
is found to be a = ( x$i + 2$j) ms −2 . Find the value of x.
(d) the magnitude of the net force on the loop is given by
(a) 4 ms −2 (b) −4 ms −2 µ 0 II1
(b − a )
(c) −2 ms −2 (d) 2 ms −2 24 ⋅ ab
(a) a circle
V. More than One Option Correct (b) a helix with uniform pitch
129. Which of the following statements are correct? (c) a helix with non-uniform pitch
(a) If a moving charged particle enters into a region of (d) a helix with uniform radius
magnetic field from outside, it does not complete a
131. A proton is fired from origin with velocity
circular path
v = v 0 $j + v 0 k$ in a uniform magnetic field B = B 0 $j.
(b) If a moving charged particle traces a helical path in a
uniform magnetic field, the axis of the helix is parallel (a) its z-coordinate can never be negative
to the magnetic field (b) its x-coordinate can never be positive
(c) The power associated with the force exerted by a (c) its x-and z-coordinates cannot be zero at the same time
magnetic field on a moving charged particle is always (d) its y-coordinate will be proportional to its time of flight
equal to zero
132. Two circular coils of radii 5 cm and 10 cm carry equal
(d) If in a region a uniform magnetic field and a uniform
electric field both exist, a charged particle moving in currents of 2 A. The coils have 50 and 100 turns
this region cannot trace a circular path respectively and are placed in such a way that their
planes as well as their centres coincide. Magnitude of
130. A charged particle moves in a uniform magnetic magnetic field at the common centre of coils is
field. The velocity of the particle at some instant (a) 8π × 10−4 T if currents in the coils are in same sense
makes an acute angle with the magnetic field. The
(b) 4 π × 10−4 T if currents in the coils are in same sense
path of the particle will be
(c) zero if currents in the coils are in opposite sense
(d) 8π × 10−4 T if currents in the coils are in opposite sense

NCERT & NCERT Exemplar Questions


NCERT 137. A 3.0 cm wire carrying a current of 10 A is placed
133. A circular coil of wire consisting of 100 turns, each of inside a solenoid perpendicular to its axis. The
radius 8.0 cm carries a current of 0.40 A. What is the magnetic field inside the solenoid is given to be 0.27 T.
magnitude of the magnetic field B at the centre of the coil? What is the magnetic force on the wire?
(a) 4 T . × 10−4 T
(b) 31 (a) 7 × 10− 2 N (b) 8.1 × 10− 2 N
(c) 2 × 10−3 T (d) 10−4 T (c) 6.4 × 10− 2 N (d) 4 × 102 N

134. A long straight wire in the horizontal plane carries a 138. Two long and parallel straight wires A and B carrying
current of 50 A in north to south direction. Give the currents of 8.0 A and 5.0 A in the same direction are
magnitude and direction of B at a point 2.5 m east of separated by a distance of 4.0 cm. Estimate the force on
the wire. a 10 cm section of wire A.
(a) 2 × 10− 6 T, south (b) 3 × 10−6 T, down (a) 5 × 10−5 N repulsive (b) 5 × 10−5 N attractive
(c) 4 × 10− 6 T, up (d) 6 T, north (c) 2 × 10−5 N repulsive (d) 2 × 10−5 N attractive
139. A closely wound solenoid 80 cm long has 5 layers of
135. A horizontal overhead power line carries a current windings of 400 turns each. The diameter of the solenoid
of 90 A in East to West direction. What are the is 1.8 cm. If the current carried is 8.0 A, estimate the
magnitude and direction of the magnetic field due to magnitude of B inside the solenoid near its centre.
the current 1.5 m below the line?
(a) 2.5 × 10− 2 T (b) 3.5 × 10− 2 T
(a) 1.2 × 10− 5 T, North (b) 1.2 × 10− 5 T, South
(c) 4.5 × 10− 2 T (d) 5 × 10− 2 T
(c) 4 T, vertically up (d) 4 T, vertically down
140. A square coil of side 10 cm consists of 20 turns and
136. What is the magnitude of magnetic force per unit carries a current of 12 A. The coil is suspended
length on a wire carrying a current of 8 A and vertically and the normal to the plane of the coil makes
making an angle of 30° with the direction of a an angle of 30° with the direction of a uniform
uniform magnetic field of 0.15 T? horizontal magnetic field of magnitude 0.80 T. What is
(a) 0.4 Nm −1 (b) 0.6 Nm −1 the magnitude of torque experienced by the coil?
(c) 4 Nm −1 (d) 6 Nm −1 (a) 1.96 Nm (b) 0.96 Nm (c) 2.0 Nm (d) 4 Nm
141. Two moving coil meters M 1 and M 2 having the 147. An electron is projected with uniform velocity along
following particulars : the axis of a current-carrying long solenoid. Which of
R1 = 10 Ω, N 1 = 30, A1 = 3.6 × 10 − 3 m 2 , B1 = 0.25 T the following is true?
R 2 = 14 Ω, N 2 = 42, A2 = 1.8 × 10 − 3 m 2 , B 2 = 050
. T (a) The electron will be accelerated along the axis
(The spring constants are identical for the two meters). (b) The electron path will be circular about the axis
Determine the ratio of voltage sensitivity of M 2 and (c) The electron will experience a force at 45° to the axis
M1 . and hence execute a helical path
(a) 4 (b) 5 (c) 6 (d) 1 (d) The electron will continue to move with uniform
142. In a chamber, a uniform magnetic field of 6.5 G (1G velocity along the axis of the solenoid
= 10 − 4 T) is maintained. An electron is shot into the 148. In a cyclotron, a charged particle
field with a speed of 4.8 × 10 6 ms −1 normal to the (a) undergoes acceleration all the time
field explain why the path of the electron is a circle. (b) speeds up between the dees because of the magnetic
Determine the radius of the circular orbit. field
. × 10 −19 C, me = 9.1 × 10 −31 kg)
( e = 16 (c) speeds up in a dee
(a) 2 cm (b) 8 cm (c) 6 cm (d) 4.2 cm (d) slows down within a dee and speeds up between dees

143. A circular coil of 30 turns and radius 8.0 cm carrying 149. A circular current loop of magnetic moment M is in
a current of 6.0 A is suspended vertically in a uniform an arbitrary orientation in an external magnetic field
horizontal magnetic field of magnitude 1.0 T. The B. The work done to rotate the loop by 30° about an
field lines make an angle of 60° with the normal of axis perpendicular to its plane is
the coil. Calculate the magnitude of the counter MB
(a) MB (b) 3
torque that must be applied to prevent the coil from 2
turning. MB
(c) (d) zero
(a) 4 Nm (b) 6 Nm (c) 3.1 Nm (d) 2.8 N m 2
150. The gyromagnetic ratio of an electron in an H-atom,
NCERT Exemplar according to Bohr model, is
144. Two charged particles traverse identical helical paths (a) independent of which orbit it is in
in a completely opposite sense in a uniform magnetic (b) negative
field B = B 0 k.
$ (c) positive
(d) increase with the quantum number n
(a) They have equal z-components of momenta
(b) They must have equal charges 151. Consider a wire carrying a steady current, I placed in a
(c) They necessarily represent a particle, anti-particle pair uniform magnetic field B perpendicular to its length.
(d) The charge to mass ratio satisfy Consider the charges inside the wire. It is known that
 e  e magnetic forces do no work. This implies that,
  +  =0
 m 1  m 2 (a) motion of charges inside the conductor is unaffected by
B, since they do not absorb energy
145. Biot-Savart, law indicates that the moving electrons (b) some charges inside the wire move to the surface as a
(velocity v) produce a magnetic field B such that result of B
(a) B is perpendicular to v (c) If the wire moves under the influence of B, no work is
(b) B is parallel to v done by the force
(c) it obeys inverse cube law (d) if the wire moves under the influence of B, no work is
done by the magnetic force on the ions, assumed fixed
(d) it is along the line joining the electron and point of
within the wire.
observation
152. Two identical current carrying coaxial loops, carry
146. A current-carrying circular loop of radius R is placed
current I in an opposite sense. A simple amperian loop
in the x-y plane with centre at the origin. Half of the
passes through both of them once. Calling the loop as C,
loop with x > 0 is now bent so that it now lies in the
(a) ∫ B ⋅ d l = mµ 0 I
y-z plane.
(a) The magnitude of magnetic moment now diminishes (b) the value of ∫ B ⋅ d l = m 2µ 0 I is independent of sense of
(b) The magnetic moment does not change C
(c) The magnitude of B at ( 0,0, z ), z > R increases (c) there may be a point on C where, B and d l are
(d) The magnitude of B at ( 0,0, z ), z >> R is unchanged perpendicular
(d) B vanishes everywhere on C
153. A cubical region of space is filled with some uniform (c) both particles gain or loose energy at the same rate
electric and magnetic fields. An electron enters the (d) the motion of the Centre of Mass (CM) is determined by
B alone
cube across one of its faces with velocity v and a
positron enters via opposite face with velocity − v. 154. A charged particle would continue to move with a
At this instant, constant velocity in a region wherein,
(a) the electric forces on both the particles cause identical (a) E = 0, B ≠ 0
accelerations (b) E ≠ 0, B ≠ 0
(b) the magnetic forces on both the particles cause equal (c) E ≠ 0, B = 0
accelerations (d) E = 0, B = 0

Answers
1. (b) 2. (c) 3. (d) 4. (d) 5. (d) 6. (c)(a) 8. (c)
7. 9. (a) 10. (a) 11. (b) 12. (b) 13. (d) 14. (a) 15. (a)
16. (b) 17. (c) 18. (d) 19. (c) 20. (b) 21. (b)(d) 23. (b) 24. (b)
22. 25. (c) 26. (c) 27. (d) 28. (d) 29. (b) 30. (a)
31. (d) 32. (c) 33. (b) 34. (b) 35. (a) 36. (c)(a) 38. (b) 39. (c)
37. 40. (c) 41. (c) 42. (c) 43. (d) 44. (c) 45. (b)
46. (c) 47. (c) 48. (a) 49. (d) 50. (d) 51. (b)(a) 53. (d) 54. (a)
52. 55. (d) 56. (d) 57. (a) 58. (a) 59. (a) 60. (c)
61. (c) 62. (c) 63. (c) 64. (d) 65. (d) 66. (a)(c) 68. (a) 69. (a)
67. 70. (c) 71. (d) 72. (d) 73. (d) 74. (c) 75. (c)
76. (d) 77. (a) 78. (d) 79. (d) 80. (a) 81. (d)(b) 83. (d) 84. (a)
82. 85. (b) 86. (a) 87. (b) 88. (b) 89. (b) 90. (a)
91. (b) 92. (b) 93. (d) 94. (d) 95. (c) 96. (c)(c) 98. (c) 99. (a)
97. 100. (a) 101. (c) 102. (b) 103. (c) 104. (b) 105. (d)
106. (a) 107. (b) 108. (c) 109. (d) 110. (d) 111. (b)(c) 113. (c) 114. (a)
112. 115. (c) 116. (c) 117. (a) 118. (d) 119. (a) 120. (c)
121. (d) 122. (b) 123. (c) 124. (b) 125. (a) 126. (b)(b) 128. (b) 129. (a,b,c
127. 130. (b,d 131. (b, 132. (a,c 133. (b) 134. (c) 135. (b)
,d) ) d) )
136. (b) 137. (b) 138. (c) 139. (a) 140. (b) 141. (d) 142. (d) 143. (c) 144. (d) 145. (a) 146. (a) 147. (d) 148. (a) 149. (d) 150. (b)

Hints and Explanations


1. (b) For proton, v ⊥ B and for electron v || B. So, force on 6. (c) When a charged particle moves in the region of magnetic
proton = q ( v × B ) = Bqv whereas, force on electron = 0. field, then force is perpendicular to the velocity and it
produces a change of direction.
2. (c) Direction of force can be found by left hand rule. As we
know the direction of F is the direction of cross product of 7. (a) Net force on the proton,
velocity v and magnetic field B, which is perpendicular to i.e., F = B q vsin θ
the plane containing v and B. = 1.5 ×1.6 ×10−19 × 2 ×107 × sin 30° = 2.4 × 10−12 N
3. (d) The Oersted found that the I 8. (c) As the electric field is switched on, positive ion will start
alignment of the magnetic needle is to move along positive x-direction and negative ion along
tangential to an imaginary circle which negative x-direction. Current associated with motion of both
has the straight current-carrying wire, B
types of ions is along positive x-direction. According to
as its centre has its plane perpendicular Fleming’s left hand rule force on both types of ions will be
to the wire as shown in figure. along negative y-direction.
4. (d) 9. (a) The total number of mobile charge carriers in wire will
be nAl. For a steady current I in this conducting wire , we
E may assume that each mobile carrier has an average drift
Current velocity vd . In the presence of an external magnetic field B,
B
the force on these carriers is given by
F = ( nAl ) q ( vd × B )
If an electron is travelling horizontally towards East and
magnetic field in vertically downward, then according to left 10. (a) According to the question,
hand rule. So, force on electron is towards North. A
F 10−10
5. (d) Magnetic field, i.e., B = = −12 = 10−3 Wb m−2
q v 10 × 105
So, according to left hand rule, magnetic field acting along
B C
Z-axis.
FAB + FBC + FCA = 0 mv
Now, r= ≥ (b − a )
FBC + FCA = 0 (Q FAB = 0 ) qB
FCA = − FBC = − F q ( b − a )B q ( b − a )B
or v≥ ⇒ v min =
11. (b) When a current carrying closed loop is placed in a m m
uniform magnetic field, F net = 0, only torque acts.
17. (c) Radius of circular path followed by a particle,
mv 9 × 10−31 kg × 3 × 107 ms −1
12. (b) r = = mv v 2 × 105
qB 1.6 × 10−19 C × 6 × 10−4 T r= = = = 16 cm
B q B q 0.05 × 2.5 × 107
= 28 × 10−2 m = 28 cm m
Frequency of an electron, 18. (d) As we know that frequency of proton,
i.e., ν = v ( 2πr ) ≈ 20 × 106 s −1 1 qB 1.6 × 10−19 × 6.28 × 10−4
f = = =
≈ 20 × 10 Hz ≈ 20 MHz
6
T 2π m 22
2× × 1.67 × 10−27
Amount of energy required 7
E = (1/ 2 )mv2 = (1/ 2 ) 9 × 10−31 kg × 9 × 1014 m2s −2 ≈ 10−19 − 4 + 27 ≈ 104 Hz
= 40.5 × 10−17 J 19. (c) As, velocity v component is along magnetic field B. So,
−16 −19
= 4 × 10 J = 2.5 keV (1 eV = 1.6 × 10 J) path is helical. Radius of a helix,
mv⊥ 1.67 × 10−27 × 2 ×106
13. (d) When a particle moves perpendicular to the magnetic r= = ⇒ r = 0.2 m
field. It has a tendency to perform circular motion in a plane Bq 0.104 × 1.6 × 10−19
perpendicular to the magnetic field. When this is coupled 2π m
with the velocity parallel to the field, then resulting Time period, T = = 2π × 10−7 s
trajectory will be a helix along the magnetic field line as Bq
shown in figure. 20. (b)
I

Y mg B
l
B

There is an upward force F of magnitude IlB. For mid-air


y

suspension, this must be balanced by the force due to gravity.


vn
v mg 0.2 × 9.8
θ mg = IlB ⇒ B = = = 0.65 T
vp
X Il 2 × 1.5
21. (b) The radius of circular path,
14. (a) As we know that radius of circle,
mv v
mv 1  2mv2  2E r= ⇒ r∝
r= =   ⇒ r= qB B
Bq 2  Bqv  Bqv
mv 2mK
Energy is same. So, r ∝ 1/ q but in case of deuteron and 22. (d) Q r = =
α-particle Bq Bq
q is same and q for proton is least rp mp K p mp K p
∴ rp > rα = rd = ⇒ 1=
rd md K d 2mp K d
mv
15. (a) Radius of path, r = K p = 2K d ⇒ K p = 2 × 50 = 100 keV
rB
ma va mb vb 23. (b) Y
Given, ra > rb ⇒ > ⇒ ma va > mb vb
qB qB E
FE
16. (b) The particle moves in a circular path of radius r in the
magnetic field. It can just enter the region x > b for r ≥ ( b − a ). v
X
Direction of B
S current FB
Y Q Z
B
When electric and magnetic fields are perpendicular to each
other.
v
So, FE = qE = qE $j, FB = qv × B
x =a x =b
P R X = q ( v$i × Bk$ ) = − qv$j (Q $i × k$ = − $j)
Therefore, force on electron is given by 30. (a)
F = q ( E − vB )$j
E σ
24. (b) Velocity of a particle, v ==
B ε0 B
∴ Time taken by electron to pass through the region,
l lε 0 B
t= =
v σ
25. (c) The cyclotron is a machine to accelerate charged particles Curl the palm of your right hand around the circular wire
or ions to high energies. The cyclotron used both electric and with the fingers pointing in the direction of the current. The
magnetic fields in combination to increase the energy of right hand thumb gives the direction of the magnetic field.
charged particles. 31. (d) Since, the coil is tightly wound, we may take each
eB circular element to have the same radius R = 10 cm = 0.1 m.
26. (c) Frequency, ν = The number of turns N = 100. The magnitude of the
2πm
magnetic field is
1 mv
KE = mv2 and radius R = µ NI 4 π × 10−7 × 102 × 1
2 eB B= 0 =
πR 2πR 2R 2 × 10−1
Here, velocity v = = = 2πRν
T/2 T = 2π × 10−4 = 6.28 × 10−4 T
m( 2πRν ) 32. (c) Magnetic field at the centre of the loop,
∴ Radius, R =
eB µ 0 NI 4 π × 10−7 × 100 × 0.4
2πmν B= = = 3.1 × 10−4 T
Magnetic field, B = 2r 2 × 8.0 × 10−2
e
1 33. (b) The magnitude of magnetic field in
Kinetic energy, K = m( v )2 = 2mπ 2 ν 2 R 2 current-carrying circular loop,
2
µ NI
27. (d) Radius in magnetic fields of circular orbit, B= 0
2R B
mV 2mE 4 π × 10−7 × 20 × 16
R= = BX =
qB qB Y
2 × 016
.
and total energy of a moving particle in a circular orbit,
= 4 π × 10−4 T
q2B 2R 2
E= 4 π × 10−7 × 25 × 18
2m BY = = 9 π × 10−4 T
For a proton entering in a region of magnetic field, 2 × 01
.
e2 × B 2 × R 2 B net = BY − B X = 5π × 10−4 T
E1 = …(i)
2 × mP = 1.57 × 10−3 T (towards west)
where, mP is the mass of proton.
34. (b) Net force on a Y
Similarly, for an α-particle moving in a uniform magnetic field current-carrying loop in a i Fm B
( 2e )2 × B 2 × R 2 uniform magnetic field is zero.
E2 = (Q mα = 4 mP ) …(ii)
2 × ( 4 mP ) Hence, the loop cannot translate.
X
So, options (c) and (d) are
Dividing Eq. (ii) by Eq. (i), we get
wrong.
E2 ( 2e )2 × B 2 × R 2 2 × mP
= × 2 From Fleming’s left hand rule,
E1 2 × ( 4 mP ) e × B2 × R2 we can see that if magnetic
E2 field is perpendicular to paper inwards and current in the
= 1 ⇒ E2 = E1 = 1 MeV loop is clockwise (as shown), the magnetic force Fm on
E1
each element of the loop is radially outwards or the loops will
28. (d) Frequency of an electron have a tendency to expand.
Bq 1 × 1.6 × 10−19
f = = = 28 GHz 35. (a) Magnetic field due to a straight current-carrying
2π m 2 × 3.14 × 9.1 × 10−31 conductor
29. (b)Q v = 3.2 × 10 ms
7 −1
⇒ B = 5 × 10 T −4 µ 0I 4 π × 10−7 × 35
B= = = 3.5 × 10−5 T
The frequency of electron, 2πr 2 × π × 0.20
qB 1.6 × 10−19 × 5 × 10−4 36. (c) Vector notation of Biot-Savart’s law,
ν= =
2πm 2 × 3.14 × 9.1 × 10−31 µ 0 Idl × r
dB =
ν = 1.4 × 107 Hz = 14 MHz 4π r3
µ 0I
37. (a) The magnitude of magnetic field, 43. (d) B at centre of a circle =
µ 0 I dl sin θ 2R
| dB | = µI
4π r2 B at centre of a square = 4 × (sin 45° + sin 45° )
l
Y 4π ⋅
2
P
µ 0I
=4 2
0.5 m 2πl
L L
Now, R = and l = (as L = 2πR = 4 l )
X 2π 4
∆x
where, L = length of wire
10−7 × 10 × 10−2 µ I πµ 0 I µ I 
i.e., |dB | = = 4 × 10−8 T ∴ BA = 0 = =π 0 
25 × 10−2 L L  L 
2⋅

As, dl × r = ∆x$i × y$j = y ∆x ( i$ × $j ) = y ∆xk$
So, the direction of the field is in the + Z-direction. µ 0I 8 2µ 0 I 8 2  µ 0 I 
BB = 4 2 = =  
µ 0 Idl × r µ Idl sin θ  L πL π  L 
2π  
38. (b) | dB | = = 0 ×  4
4π r 3
4π r2
If point lies on the conductor, then θ = 0° or 180° and ∴ B A : BB = π 2 : 8 2
sin θ = 0, then dB = 0. Hence, the magnetic field induction at 44. (c) Magnetic field at centre of current-carrying coil,
any point on the conductor itself is zero. µ 0I
µ πi 22 × 12 Bc = …(i)
39. (c) B = 0 = 10−7 × = 1.9 × 10−4 T 2r
4π R 7 × 2 × 10−2 Magnetic field at axial point due to a current-carrying coil at
Thus, B is 1.9 × 10−4 T normal to the plane of the paper distance of r,
going into it. d=r
µ 0 Ir2 µ 0 Ir2
41. (c) Magnetic field at any point lying on the current-carrying Bc = ⇒ Ba = …(ii)
straight conductor is zero. 2( r + d )
2 2 3/ 2
2( 2r2 )3 / 2
Here, H1 = magnetic field at M due to current in PQ. Bc µ 0 I 2( 2r2 )3 / 2
H 2 = magnetic field at M due to QR + magnetic field Now, = × =2 2
Ba 2r µ 0 Ir2
at M due to QS + magnetic field at M due to PQ.
H 3 H 2 Bc : Ba = 2 2 : 1
= 0 + 1 + H1 = H1 ⇒ 1 =
2 2 H2 3 45. (b) Consider two amperian loops of
42. (c) radius a/ 2 and 2a as shown in the a/2
diagram. Applying Ampere’s circuital law
R for these loops, we get
2a
θ
θ
∫ B ⋅ dL = µ 0 I enclosed
r For the smaller loop,
2
a I  a 1 µ I
⇒B × 2π = µ0 × × π   = µ 0I × = 0
2π 2 πa 2  2 4 4
Angle subtended at centre by any of side =
n µ I a
2π π ⇒ B′ = 0 , at distance from the axis of the wire.
⇒ 2θ = ;θ = 4 πa 2
n n Similarly, for bigger amperian loop
Field due to one side, B ′ × 2π ( 2a ) = µ 0 I
µ I
B1 = 0 (sin θ + sin θ ) (total current enclosed by Amperian loop is 2.)
4 πr µ I
π π ⇒ B′ = 0
But, r = R cos θ = R cos and sin θ = sin 4 πa
n n
µ 0I π µ I π at distance 2a from the axis of the wire.
∴ B1 = × 2sin = 0 tan B µ I 4 πa
π n 2πR n So, ratio of = 0 × =1
4 π R cos B ′ 4 πa µ 0 I
n
and so field on n sides at centre will add up to form net field 46. (c) For a cylindrical conductor for r < a , B ∝ r and for
µ nI π r > a , B ∝ (1/ r ). So, graph (c) is correct.
Bcentre = 0 tan .
2πR n
µ 0 I1 µ I
47. (c)
Q B1 = and B 2 = 0 2
2π d 2π d
µ0 1 2
B net = B12 + B22 ⇒ B net = ( I1 + I 22 )1 / 2
P 2π d
54. (a) I1 = 4 A , I 2 = 5 A ,d = 4 × 10−2 m, l = 0.20 m
Force on a current carrying wire,
µ 0 I1 I 2 l 4 π × 10−7 × 4 × 5
48. (a) The number of turns per unit length is F = = × 0.20
2πd 2 × π × 4 × 10−2
500
n= = 1000 turns/m = 2 × 10−5 N towards B
0.5
The length ( l ) = 0.5 m and radius ( r ) = 0.01 m. 55. (d) According to given information, following figure can be
Thus, l/ a = 50 i.e., l >> a. drawn, which shows that direction of magnetic field is along
the direction of motion of charge, so net force on it is zero.
Hence, we can use the long solenoid formula, namely,
B = µ 0 nI = 4 π × 10−7 × 103 × 5 = 6.28 × 10−3 T
49. (d) By the formula, v
−3 −7
B = µ 0 nI or 20 × 10 = 4 π × 10 × 2000 × I q
−3
20 × 10 d
or I= −7
⇒ I ~
− 8A
4 π × 10 × 2000 d/2 d/2

50. (d) For toroid, applying Ampere’s circuital law, 56. (d) Consider the given figure,
µ NI Y
B C
B ( 2πr ) = µ 0 NI ⇒ B = 0
2πr
where, B = magnetic field of a toroid I i L

N = number of turns of toroidal coil


r = radius of toroid X A D
L/2 L
0.25 + 0.26
51. (b) Mean radius, rm = = 25.5 × 10−2 m µ 0 IiL µ 0 IiL
2 FBA = ⇒ FCD =
N 3500  L  3L
n= = 2π   2π  
2π rm 2π × 25.5 × 10−2  2  2
Magnetic field of a toroid Therefore, net force on the loop ABCD will be
−7 3500 −2 µ 0 IiL  1 1 
B = µ 0 nI = 4 π × 10 × × 11 = 3 × 10 T Floop = FBA − FCD = −
2π × 25.5 × 10−2 2π  ( L/ 2 ) ( 3L/ 2 )
52. (a) The force acting per unit length, 2 µ 0 iI
Floop =
F µ 0 2I1 I 2 3π
= F µ 0 I1 I 2
l 4π d 57. (a) The force acting per unit length of a wire, =
When d = 1m, l = 1m, I1 = I 2 = 1 A l 2π d
Then, F = 2 × 10−7 Nm−1 Also I1 and I 2 are opposite directions.
∴ They repel each other, F = 1.2 Nm−1
53. (d) A
I1 58. (a) Current flows in same direction.
O F µ 0I 2
C D ∴ Wires attract each other and =
I2 l 2π d
F AC F A C B
B 59. (a) = BC
l l
The point P is lying at a distance d along the Z-axis.
µ 0 2 × 15 × 50 µ 0 2 × 50 × 10
⋅ = × 15 A 50 A 10 A
P
B1 4π x 4π 15 − x
90° 15 10
=
x (15 − x ) x 15 – x
B2 225 − 15x = 10x ⇒ 25x = 255 or x = 9 cm
60. (c) Since, B is uniform only torque acts on a 65. (d) For parallel M is stable and for anti-parallel is unstable.
current-carrying loop. 66. (a) The torque acting on the coil | τ | = | m × B | = mB sin θ.
As, τ = M × B ⇒ | τ | = | M|| B | sin θ Here the circular coil is placed normal to the direction of
For orientation shown in (b) θ = 0° , τ = 0 magnetic field then the angle between the direction of
(Q stable equilibrium) magnetic moment ( m ) and magnetic field ( B ) is zero, then
∴ and for (d) θ = π , τ = 0 (Q unstable equilibrium) τ = mB sin θ = mB sin 0 = 0 ⇒ τ = 0
61. (c) As we know, potential energy i.e., 67. (c) For a loop at a point on its axis distant x from centre,
U = − MB cosθ, where θ = angle between normal to the µ 0 IR 2
B=
plane of the coil and direction of magnetic field. 2 ( x2 + R 2 )3 / 2
62. (c) As, we know that initial magnetic moment = µ 1 = iL 2
µ 0R 2 I µ 0 IA
For x >> R, B= ⇒ B=
i 2x 3
2πx 3
L Hence, ~ µ 0 m = µ 0 × 2m
B−
2πx 3 4 π x3
68. (a) I

L/2 L
µ1
L L/2
µ1 = iL2
µ 0I
M For a single coil, magnetic field, Bs = T …(i)
µ2 = M √2 2R
I
M
After folding the loop,
I
M = magnetic moment due to each part
 L iL2 µ 1 For two coils magnetic field,
= i × L= =
 2 2 2 µ 0I
B T = B12 + B22 = 2 …(ii)
µ1 µ 2R
⇒ µ2 = M 2 = × 2= 1
2 2 On comparing Eq. (i) and Eq. (ii), we get
BS 1
63. (c) Torque acting on equilateral triangle in a magnetic field =
B is BT 2
L
69. (a) Magnetic moment of a current-carrying loop is
m = NIA
S N
B l where, N is number of turns, I is current flowing on a loop
90° and A is area of cross-section of loop.
70. (c) Here, N = 150 , R = 12 cm = 12 × 10−2 m , I = 2A
M N µ 0 NI 2π × 10−7 × 150 × 2
O B= =
Normal 2R 12 × 10−2
τ = M × B, τ = i AB sin θ ( iA = M ) = 1.57 × 10−3 T = 15.7 × 10−4 T = 15.7 G
Area of ∆ LMN , 71. (d) Each segment experiences a force, so it tends to
3 2 assume a circular shape.
A= l and θ = 90° ( l = sides of triangle )
4 72. (d) Work done = MB(1 − cos θ ),θ = 0° so work done is zero.
Substituting the given values in the expression for torque, 73. (d) Magnetic field at the centre of circular loop,
we have
µ 0 2πI µ 0 I 2Br
3 2 3 2 B= = ⇒I =
τ=i× l B sin 90° = il B (Qsin 90° = 1) 4π r 2r µ0
4 4
1/ 2 Also, A = πr or r = ( A / π )
2 1/ 2
 τ 
Hence, l = 2   2Br 2BA  A 
1/ 2
 3 Bi  Magnetic moment, M = IA = A= × 
µ0 µ0  π
64. (d) Torque on current loop in a field is τ = m × B.
2BA 3 / 2
where, m is magnetic moment of a current carrying loop B is =
magnetic field. µ 0 π1 / 2
e–
74. (c) The uniform circular motion i 81. (d) Uniform field is made radial by cutting pole pieces radially.
of the electron constitutes a r 82. (b) The deflection in a moving coil galvanometer,
current in anti-clockwise. The NAB
direction of the magnetic moment φ= ⋅ I or φ ∝ N , where N is number of turns in a
is into the plane of the paper and +Ze µe k
is indicated separately by ⊗. coil, B is magnetic field, A is area of cross-section.
83. (d) The deflecting torque acting on the coil
75. (c) T is the time period of revolution. Let r be the orbital
radius of the electron, and v the orbital speed. Then, τ deflection = NIAB
2π r 2 86. (a) Current sensitivity of the galvanometer deflection per
T= ,I = unit ampere
v T
φ NBA
On substituting, we have I = ev / 2π r. ⇒ =
I k
76. (d) Magnetic field on a current-carrying in a circular orbit
µ I 87. (b) Current sensitivity of all galvanometer
i.e., B = 0 φ NBA
2R = Si =
I k
B × 2R 12.56 × 2 × 5.2 × 10−11
∴ I= = −7
= 1.04 × 10−3 A Hence, to increase S i , k must be decreased.
µ0 4 π × 10
Nm
e 88. (b) Spring constant, k = 10−8
77. (a) For n = 1, for an H2 atom, µ l min = h rad
4πme
N = 100, A = 1 cm2 = (1 × 10−2 )2 m2 = 1 × 10−4 m2 B = 5 T,
1.60 × 10−19 × 6.63 × 10 − 34
= = 9.27 × 10−24 Am2 NBA 100 × 5 × 1 × 10−4
4 × 3.14 × 9.11 × 10−31 ∴ Si = = = 5 × 106 A −1
k 10−8
where, the subscript min stands for minimum. This value is Ammeter
called the Bohr magneton. 89. (b) As potential is same for parallel
combination. S
78. (d) There will be a magnetic moment, usually denoted by µ i
associated with this circulating current. For ammeter, S ( I − I g ) = GI g
I I–Ig
Its magnitude is µ i = Iπr2 = evr / 2. G Ig G
Shunt resistance, S = Ig I
Multiplying and dividing the right hand side of the above I − Ig
expression by the electron mass me , 15 × 4 × 10−3
e e = = 10 mΩ
We have µl = ( me vr ) = l 6 − 4 × 10−3
2m e 2m e
90. (a) Minimum current measured by A → 0.3 mA
Here, l is the magnitude of the angular momentum of the
electron about the central nucleus (orbital angular momentum). by B → 0.5 mA
Vectorially, 91. (b) According to ammeter circuit, we get ( I − I g ) S = I g G
e
µl = − l where, G is resistance of galvanometer.
2me
The negative sign indicates that the angular momentum of the 92. (b) To convert a galvanometer into ammeter, a low resistance
electron is opposite in direction to the magnetic moment. is connected in parallel with the coil of galvanometer.
RG rs
79. (d) A moving coil galvanometer is a sensitive instrument 93. (d) R =
which is used to measure a deflection when a current flows RG + rs
through its coil. Here, RG = 70 Ω , rs = 0.03 Ω
70 × 0.03
80. (a) When a current flows through the coil, a torque acts on it. ∴ R= = 0.02998 = 0.03 Ω
This torque is given by τ = NIAB 70 + 0.03
S 15
. Si 3
where, the symbols have their usual meaning. Since, the 94. (d) S V = i ⇒ S V′ = = SV
field is radial by design, we have taken sin θ = 1in the above R 2R 4
expression for the torque. The magnetic torque NIAB tends 1 − 3/ 4
to rotate the coil. % decrease in voltage sensitivity = × 100 = 25%
1
A spring S P provides a counter torque kφ that balances the = (1/ 4 ) S V
magnetic torque NIAB; resulting in a steady angular
deflection φ. In equilibrium kφ = NIAB 95. (c) Voltage sensitivity is deflection per unit voltage
where, k is the torsional constant of the spring, i.e., the φ NBA  NBA  1
= =  ×
restoring torque per unit twist. The deflection φ is indicated V kR  k  R
on the scale by a pointer attached to the spring. We have
96. (c) To convert a galvanometer into voltmeter, a high
 NAB 
φ=  I resistance in series is to be connected with galvanometer.
 k 
97. (c) For voltmeter circuit, 105. (d) The force experienced by a charged particle in a
( I − I g ) ⋅ RL = I g (G + S ) magnetic field is given by F = q ( v × B ), which is
where, G and S, are resistance of galvanometer and shunt. independent of mass. As qv and B are same for both the
electron and proton, both will experience same force.
98. (c) G = 12 Ω and I g = 2.5 × 10−3 A
106. (a) When a charged particle moves in region of a
GI g 12 × 2.5 × 10−3 perpendicular magnetic field, the magnetic force is
For ammeter, S = = −3 perpendicular to the velocity of the particle. So, no work is
I − Ig 7.5 − (2.5 × 10 )
done and no change in the magnitude of the velocity.
1 1 1
⇒ S = 4 × 10−3 Ω ⇒ = + 107. (b) If velocity has Y
R ammeter G S component along B, this
SG
⇒ Ra = = 4.0 × 10−3 Ω component remains v||
S +G unchanged as the motion
X
along the magnetic field B
For a galvanometer as voltmeter,
will not be affected by q
V 10.0
R= −G= − 12 = 3988 Ω the magnetic field. The
Ig 2 ⋅ 5 × 10−3 motion in a plane
Z Pitch Radius

∴ RV = Voltmeter’s resistance perpendicular to B is as before a circular one, thereby


producing a helical motion.
RV = R + G = 4000 Ω = 4 × 103 Ω
108. (c) Charged particle covers semicircular path in T / 2 s.
Ra 4 × 10−3 2π m
Ratio, = = 10−6 T=
RV 4 × 103 Bq
99. (a) R where, B is magnetic field and m is mass of electron.
G 109. (d) Any charge in uniform circular motion would have an
Ig
associated magnetic moment. This dipole moment is labelled
For voltmeter circuit, we get as the orbital magnetic moment. Hence, the subscript l in µ l .
As, Ig (R + G ) = V Besides the orbital moment, the electron has an intrinsic
V 18 magnetic moment, which has the same numerical value. It is
So, R= −G= − 12 = 5988 Ω in series
Ig 3 × 10−3 called the spin magnetic moment. But we hasten to add that
it is not as though the electron is spinning. The electron is an
NBA elementary particle and it does not have an axis to spin
100. (a) As, we know current sensitivity S i =
k around like a top or our Earth. Nevertheless it does possess
NBA this intrinsic magnetic moment. The microscopic roots of
and voltage sensitivity, S V = magnetism in iron and other materials can be traced back to
kR
Si Si this intrinsic spin magnetic moment.
⇒ = R (or G ) ⇒ = SV NBA
SV G 110. (d) Current sensitivity, S i =
k
101. (c) Cyclotron is utilised to accelerate the positive ion. NBA
Be and voltage sensitivity, S V =
Cyclotron frequency is given by ν = . kR
2πm
So, when S i is increased by increasing number of turns N ,
It means cyclotron frequency does not depend upon velocity. length of wire used also increases and so, R increases. Hence,
2πm  m  m S V may remain same or decrease whereas S i increases.
102. (b) Time period, T = as   = 2   ⇒ Tα = 2Tp
Bq  qα  q p 111. (b) If v ⊥ B, then path is a circular motion and if v has a
Also, T ∝m component along B, then path will be helical.
But, Tα = 4Tp which is not the case. 112. (c) Radius of a circle
mv × × ×
103. (c) Time taken is independent of velocity and radius of r= ⇒ r ∝ m ⇒ ve < v p B
path. However, maximum velocity will be given by Bq × × ×
qBR
v max = and time taken to complete one revolution
m × × ×
2πm B
where, R is radius of Dee’s. T= ⇒ T ∝m × × ×
Bq
104. (b) The magnetic field due to solenoid having n number of
turns/metre and carrying current I is But, mass of electron ( me ) < mass of proton ( mp )
µ nI So, Te < Tp
B = 0 (cos α − cos β )
2 113. (c) In a velocity selector, where Fe and FB are electric and
It is obvious that magnetic field is independent of length and magnetic field,
area. Also magnetic field is uniform inside the solenoid.
Case I Fe = FB ⇒ v = E / B 120. (c) A. Torque on loop, τ = m × B
Case II Fe < FB ⇒ v > E / B where, m is magnetic moment of a current-carrying loop
Case III Fe > FB ⇒ v < E / B and B is magnetic field.
B. Torque on loop when m is either parallel or anti-parallel
114. (a) Difference between Coulomb’s force and Biot-Savart’s to B, i.e., τ = m × B vanishes.
forces are
C. Magnetic moment of loop, m = NIA, where I is
I. The electrostatic field is along the displacement vector current and A is area of cross-section of wire.
joining the source and the field point. The magnetic field
D. Torque on loop when m is perpendicular to B.
is perpendicular to the plane containing the displacement
vector r and the current element Idl. τ = m × B = mB sin 90° = mB
II. There is an angle dependence in the Biot-Savart’s law 121. (d) Scale
which is not present in the electrostatic case. In the
magnetic field at any point in the direction of Idl is zero.
Along this line, θ = 0, sin θ = 0, then | dB | = 0. Pointer Permanent magnet
115. (c) As two coaxial solenoids 1 and 2 of the same length are
set to be inside the other. So, net magnetic field, Coil
i.e., B net = B1 − B2 ⇒ B1 − B2 = 0
B1 = B2 ⇒ B ∝ ni
Sp
So, n1 i1 = n2 i2 or n1 = n2 and i1 = i2 N S

116. (c) I Galvanometer is a very sensitive device, it gives a full


scale deflection for a current of the order of µΑ.
II For measuring currents, the galvanometer has to be Soft iron core
connected in series and as it has a large resistance, this will Uniform radial magnetic field
change the value of the current in the circuit. A. Core increases field strength.
117. (a) A. Since, E = 0 and B ≠ 0 so path will be straight line. If B. Pole pieces produce radial field.
velocity is parallel to B, or path will be circular if v ⊥ B , C. Spring produce restoring torque.
or path will be helical (with uniform pitch) if v is at D. Coil produces deflecting torque.
same other angle to B. mv m ( 2v0 )
122. (b) r0 = ⇒ r′ = = 2 r0
Hence, A → 1, 3 qB qB
B. Since, E ≠ 0 and B = 0. So, path will be straight line 2πm
parellel to E or parabola otherwise. 123. (c) T = → independent of velocity.
qB
Hence, B → 1, 2
C. E ≠ 0, B ≠ 0, E || B 124. (b) As F ⊥ B
Helical path with non-uniform pitch. Hence, a ⊥ B
Hence, C → 4 ∴ a ⋅ B = 0 ⇒ ( xi$ + 2$j) ⋅ ( 2i$ + 4 $j ) = 0
v 2x + 8 = 0 ⇒ x = −4 ms −2
125. (a) Force acting on a horizontal table F = IlB sin θ
90°
So, the force per unit length is
F
B f = = IB sin θ
D. Straight line path if v × B = E l
When the current is flowing from East to West, then
Hence, D → 1
θ = 90°, so f = IB sin 90° = 1 × 3 × 10− 5 × 1 = 3 × 10−5 Nm
−1
B
126. (b) When the current is flowing from South to North, then
E θ = 0°, So, f = IB sin 0° ⇒ f = 0
Hence, there is no force on the conductor.
v 127. (b) B
A
119. (a) A. Lorentz force F = q ( E + v × B ) a
B. Gauss’ law, ∫ E ⋅ dA = q/ ε 0 O 30°
D
µ 0 idl × r b
C. Biot-Savart’s law, dB = = C
4π r3
D. Coulomb’s law, F =
1 q1 q2 Net magnetic field at O, B AB + BBC + B CD + BDA
4 πε 0 r2
µ0 I π µ I π µ I µ 0 2I 2 × 50
⇒ B = 0+ × + 0− 0 × = 0 (b − a ) B= ⋅ = 10− 7 ⋅ = 4 × 10− 6 T, upward direction
4a 6π 4b 6π 24 ab 4π r 2.5
128. (b) As, AD and BC are parallel to the field of I1 . So, force 135. (b) Overhead line
90 A
on them is zero.
West East
129. (a,b,c,d) Options (a) and (b) are theoretical facts. As in case 1.5 m
of moving charged particle in magnetic field Fmag ⊥ v,
hence power associated will be zero (option (c) is correct). P
If both the electric and magnetic fields exist : If B || E, the The magnitude of magnetic field,
path of the charged particle will be helical. If B || E, the
µ 2I 10− 7 × 2 × 90
radius of the charged particle will not be constant. Hence, B= 0⋅ = = 1.2 × 10− 5 T
the path will not be circular (option (d) is correct). 4π r 1.5
The direction of B will be perpendicularly upward.
130. (b,d) If a particle moves in a magnetic field in helical path,
the plane of the helical path will be inclined at an acute 136. (b) According to the question, I = 8 A, θ = 30°, B = 0.15 T,
angle with the magnetic field. l = 1m
Plane of I
I helical path
θ 30°
B

Hence, options (b) and (d) are correct. The magnitude of magnetic force
131. (b, d) Velocity of proton makes an angle of 45° with the F = I ( l × B ) = I l B sin θ
direction of magnetic field. Therefore, the path of the proton 8 × 0.15
= 8 × 1 × 0.15 × sin 30° = = 0.6 Nm −1
is a helix. The plane of the circle of this helix is the plane 2
formed by negative X and positive Z-axis. Therefore,
137. (b) I
x-coordinate can never be positive. Further, x and
z-coordinates will become zero simultaneously after every
B
pitch and y-coordinate of the proton at any time t is
y = v0 t , i.e., y ∝ t
Here, the angle between the magnetic field and the direction
µ NI
132. (a,c) Using B = 0 ⇒ | B1 | = | B 2 | = 4 π × 10−4 T of flow of current is 90°.
2R Given, l = 3 cm = 3 × 10− 2 m, I = 10 A, B = 0.27 T,
If current in same sense, B net = 8π × 10−4 T The magnitude of magnetic force on the wire,
If current in opposite sense, B net = 0 F = IlB sin 90° = 10 × 3 × 10− 2 × 0.27 × sin 90°
133. (b) Here, n = 100, r = 8 cm = 8 × 10− 2 m = 8.1 × 10− 2 N
138. (c) Given, I1 = 8 A, I 2 = 5 A and r = 4 cm = 0.04 m
A B
O
r
8A 5A
B
4 cm
and I = 0.40 A F
The magnetic field B at the centre,
Force per unit length on two parallel wire carrying-current
µ 2πIn 10− 7 × 2 × 3.14 × 0.4 × 100
B= 0⋅ = = 3.1 × 10− 4 µ 0 2I1 ⋅ I 2 10− 7 × 2 × 8 × 5
4π r 8 × 10− 2 F = ⋅ = = 2 × 10− 4 N
T 4π r 0.04
134. (c) Given, I = 50 A and r = 2.5 m The force on A of length 10 cm is F ′ = F × 0.1
North (Q 1m = 100 cm)
⇒ F ′ = 2 × 10− 4 × 0.1 = 2 × 10− 5 N
2.5 m East 139. (a)
r P
50 A B

South ∴ The total number of turns, N = 400 × 5 = 2000


The magnitude of magnetic field,
2000 (i) speeds up between the dees because of the oscillating
and number of turns/length, n = = 2500
0.8 electric field and
The magnitude of magnetic field inside the solenoid (ii) speed remains the same inside the dees because of the
magnetic field but direction undergoes change
B = µ 0 nI = 4 × 3.14 × 10− 7 × 2500 × 8 = 2.5 × 10− 2 T
continuously.
The direction of magnetic field is along the axis of solenoid.
149. (d) The rotation of the loop by 30° about an axis
140. (b) The magnitude of torque experienced by the coil, perpendicular to its plane make no change in the angle made
τ = NI AB sin θ by magnetic moment with the direction of magnetic field,
therefore, the work done to rotate the loop is zero.
= 20 × 12 × (10 × 10− 2 )2 × 0.80 × sin 30°
150. (b) If L is the magnitude of the angular momentum of the
2.4 × 0.80
τ = 2.4 × 0.80 sin 30° = = 0.96 Nm electron about the central nucleus (orbital angular
2 momentum). Vectorially,
141. (d) Using the formula of voltage sensitivity, e
µl = − L
NAB 2me
V =
kR where, µ l is the magnetic moment.
Vs 2 n B A .k R 42 × 0.50 × 1.8 × 10− 3 × k × 10 The negative sign indicates that the angular momentum
∴ = 2 2 2 1 1 =
Vs 1 k2 . R2 . n1 B1 A1 k × 14 × 30 × 0.25 × 3.6 × 10− 3 of the electron is opposite in direction of the magnetic
moment.
=1
mv 151. (b,d) Magnetic forces on a wire carrying a steady current, I
142. (d) r = [Here, F = q ( v × B ) = e ( v × B ) = eV B sin 90° ] placed in a uniform magnetic field B, perpendicular to its
Bq length is given by
mv2 F = IlB
evB sin 90° =
r
The direction of force is given by Fleming's left hand rule
mv 9.1 × 10−31 × 4.8 × 106 and F is perpendicular to the direction of magnetic field B.
⇒ r= =
eB × 1 1.6 × 10−19 × 6.5 10−4 Therefore, work done by the magnetic force on the ions is
zero.
= 4.2 × 10− 2 m = 4.2 cm
152. (b,c) Applying the Ampere’s circuital law, we have
143. (c) Magnitude of torque acting on the current-carrying coil
due to the magnetic field, ∫cB ⋅ d l = µ 0 ( I − I ) = 0 (because current is in opposite
sense.)
Also, there may be a point on C where B and dl are
60° perpendicular and hence, ∫ B ⋅ d l = 0
c
B
153. (b,c,d) The magnetic force F = q ( v × B ) on charged particle
is either zero or F is perpendicular to v (or component of v)
which in turn revolves particles on circular path with
τ = nIAB sin θ uniform speed. In both the cases, particles have equal
= 30 × 6 × π (0.08) 2 × 1 × sin 60° accelerations.
3 Both the particles gain or loss energy at the same rate as
= 30 × 6 × 3.14 × 0.08 × 0.08 × = 3.133 Nm both are subjected to the same electric force (F = q E) in
2
opposite direction.
144. (d) For given pitch, d corresponds to charged particle, we
Since, there is no change of the Centre of Mass (CM) of the
have
particles , therefore the motion of the Centre of Mass (CM)
q 2πvcos θ  e  e
= = constant ⇒   +   = 0 is determined by B alone.
m qB  m 1  m 2
154. (a,b,d) Here, force on charged particle due to electric field
Note Consider e in place of q in solution. FE = qE.
145. (a) Magnetic field associated with moving charge Force on charged particle due to magnetic field,
Fm = q ( v × B )
µ v× r
B= q 2 Now, FE = 0if E = 0and Fm = 0if sin θ = 0or θ° = 0° or 180°
4π r
So, B⊥v Hence, B ≠ 0 .
Also, E = 0 and B = 0 and the resultant force
147. (d) F = qvB sin θ = 0 as θ = 0 or π
q E + q ( v × B ) = 0. In this case, E ≠ 0 and B ≠ 0.
148. (a) The charged particle undergoes acceleration as

You might also like